Last active
October 23, 2024 13:24
-
-
Save Gro-Tsen/bc61667d362994f4a49f5a613390de24 to your computer and use it in GitHub Desktop.
This file contains bidirectional Unicode text that may be interpreted or compiled differently than what appears below. To review, open the file in an editor that reveals hidden Unicode characters.
Learn more about bidirectional Unicode characters
\documentclass[12pt,a4paper]{article} % -*- coding: utf-8 -*- | |
\usepackage[a4paper,margin=1.5cm]{geometry} | |
\usepackage[english]{babel} | |
\usepackage[utf8]{inputenc} | |
\usepackage[T1]{fontenc} | |
\usepackage{times} | |
\usepackage{amsmath} | |
\usepackage{amsfonts} | |
\usepackage{amssymb} | |
\usepackage{amsthm} | |
% | |
\usepackage{mathrsfs} | |
%\usepackage{bm} | |
%\usepackage{stmaryrd} | |
\usepackage{wasysym} | |
\usepackage{url} | |
\usepackage{graphicx} | |
\usepackage[usenames,dvipsnames]{xcolor} | |
\usepackage{tikz} | |
\usetikzlibrary{matrix,arrows,decorations.markings} | |
%\usepackage{hyperref} | |
% | |
% | |
% | |
\theoremstyle{definition} | |
\newtheorem{comcnt}{Anything} | |
\newcommand\thingy{% | |
\refstepcounter{comcnt}\medbreak\noindent\textparagraph\textbf{\thecomcnt.} } | |
\newtheorem{thm}[comcnt]{Theorem} | |
\newtheorem{prop}[comcnt]{Proposition} | |
\newtheorem{lem}[comcnt]{Lemma} | |
\newtheorem{ques}[comcnt]{Question} | |
% | |
% | |
% | |
\newcommand{\apart}{\mathrel{\#}} | |
% | |
% | |
% | |
\begin{document} | |
\pretolerance=8000 | |
\tolerance=50000 | |
\thingy We work in a constructive math setting in the absence of any | |
form of Choice (except Unique Choice; e.g., a topos with a natural | |
numbers object, or in $\mathsf{IZF}$). We denote by $\mathbb{R}$ the | |
set of Dedekind reals, and by $x\apart y$ the relation $|x-y|>0$ | |
(equivalently, “$x<y$ or $x>y$”); recall that $x=y$ means | |
$\neg(x\apart y)$ (but $x\apart y$ is \textit{a priori} stronger than | |
$\neg(x=y)$ which is just $\neg\neg(x\apart y)$). | |
\thingy\label{wlpo-and-friends} We consider the following principles, | |
the implications between which we wish to investigate: | |
\begin{itemize} | |
\item “$\mathbf{DEC}(\mathbb{R})$”: There exists subsets $A,B | |
\subseteq \mathbb{R}$ such that $A$ and $B$ are inhabited, $A \cap B | |
= \varnothing$ and $A \cup B = \mathbb{R}$. | |
\item “$\mathbf{WLPO}_{\mathbb{R}}$”: Every $c\in \mathbb{R}$ | |
satisfies $c=0$ or $\neg(c=0)$. | |
\item “$\mathbf{WLPO}_{\mathrm{seq}}$”: Every binary sequence $(p_n)$ | |
(i.e., element of $\{0,1\}^{\mathbb{R}}$) satisfies $\forall | |
n.(p_n=0)$ or $\neg\forall n.(p_n=0)$. | |
\end{itemize} | |
\thingy\label{dec-vs-chi-function} Note that | |
$\mathbf{DEC}(\mathbb{R})$ is equivalent to the existence of a | |
function $\chi\colon\mathbb{R}\to\{0,1\}$ such that $\exists | |
a\in\mathbb{R}.(\chi(a)=0)$ and $\exists b\in\mathbb{R}.(\chi(b)=0)$. | |
Furthermore, replacing $\chi$ by $t \mapsto \chi(a+(b-a)t)$, we can | |
assume that in fact $\chi(0)=0$ and $\chi(1)=1$. We will generally | |
work with this form. | |
\thingy\label{analytic-wlpo-implies-sequential-wlpo} It is a standard | |
fact that $\mathbf{WLPO}_{\mathbb{R}}$ implies | |
$\mathbf{WLPO}_{\mathrm{seq}}$. This will be reproved incidentally in | |
the course of what follows, but a shorter argument goes as follows. | |
If $(p_n)$ is a binary sequence, we define a real sequence $z_n := | |
\sum_{k=0}^n p_k\cdot 2^{-k}$. Clearly this sequence is Cauchy with | |
modulus $|z_m-z_n| \leq 2^{-\min(m,n)}$, so it has a limit $z$. If | |
there is $n$ such that $p_n = 1$, then $z \geq 2^{-n}$, so $z\apart | |
0$; so contrapositively, $z=0$ implies that $(p_n)$ is | |
identically $0$. But conversely, if $(p_n)$ is identically $0$ then | |
trivially $z=0$. This shows that $(p_n)$ is the zero sequence if and | |
only if $z=0$. Now $\mathbf{WLPO}_{\mathbb{R}}$ tells us that $z=0$ | |
or is not. So we conclude that $(p_n)$ is the zero sequence or it is | |
not. This gives $\mathbf{WLPO}_{\mathrm{seq}}$.\quad\qedsymbol | |
\begin{thm}\label{dec-r-equivalent-to-analytic-wlpo} | |
$\mathbf{DEC}(\mathbb{R})$ is equivalent to | |
$\mathbf{WLPO}_{\mathbb{R}}$ | |
\end{thm} | |
The implication $\mathbf{WLPO}_{\mathbb{R}} \; \Longrightarrow \; | |
\mathbf{DEC}(\mathbb{R})$ is trivial. Indeed, | |
$\mathbf{WLPO}_{\mathbb{R}}$ asserts precisely that $\mathbb{R}$ is | |
the disjoint union of $\{0\}$ and $\mathbb{R}\setminus\{0\} := \{t : | |
\neg(t=0)\}$. | |
The converse follows immediately from lemmate | |
\ref{dec-r-implies-sequential-wlpo} and \ref{dec-r-and-sequential-wlpo-imply-analytic-wlpo}. | |
\begin{lem}[Mohammad Tahmasbi]\label{dec-r-implies-sequential-wlpo} | |
$\mathbf{DEC}(\mathbb{R})$ implies $\mathbf{WLPO}_{\mathrm{seq}}$ | |
\end{lem} | |
\begin{proof} | |
Assume $\chi\colon\mathbb{R}\to\{0,1\}$ satisfies $\chi(0)=0$ and | |
$\chi(1)=1$. | |
Define real sequences $(u_n)$ and $(v_n)$ by induction as follows: put | |
$u_0=0$ and $v_0=1$, and then : | |
\begin{itemize}\setlength\itemsep{0pt} | |
\item when $\chi(\frac{u_n+v_n}{2})=0$ let $u_{n+1} = | |
\frac{u_n+v_n}{2}$ and $v_{n+1} = v_n$, | |
\item when $\chi(\frac{u_n+v_n}{2})=1$ let $u_{n+1} = u_n$ and | |
$v_{n+1} = \frac{u_n+v_n}{2}$. | |
\end{itemize} | |
Obviously we have $\chi(u_n)=0$ and $\chi(v_n)=1$ for all $n$ by | |
induction. Furthermore, $(u_n)$ and $(v_n)$ satisfy $0\leq u_m\leq | |
u_n\leq v_n\leq v_m\leq 1$ when $m\leq n$, and $|u_n-v_n| \leq | |
2^{-n}$. In particular, they are Cauchy with modulus $|u_m-u_n| \leq | |
2^{-\min(m,n)}$ and $|v_m-v_n| \leq 2^{-\min(m,n)}$, and they have the | |
same limit. Call $w$ this common limit: we have $|w-u_n| \leq 2^{-n}$ | |
and $|w-v_n| \leq 2^{-n}$. | |
We can assume without loss of generality that $\chi(w)=0$, the case | |
$\chi(w)=1$ being entirely symmetric. | |
Now let $(p_n)$ be a binary sequence. We wish to show that $(p_n)$ is | |
the zero sequence or it is not. | |
Define a real sequence $(z_n)$ as follows: | |
\begin{itemize}\setlength\itemsep{0pt} | |
\item when $p_k=0$ for all $0\leq k\leq n$, let $z_n = w$, | |
\item when there exists an $0\leq k\leq n$ such that $p_k=1$, let $z_n | |
= v_k$ where $k$ is the smallest such $k$. | |
\end{itemize} | |
(In other words, $z_n$ is constantly $w$ until the first $m$ where | |
$p_m$ takes the value $1$, at which point and after which $z_n$ equals | |
$v_m$.) This sequence is well-defined. | |
We claim that $|z_m-z_n| \leq 2^{-\min(m,n)}$. Indeed, with $m\leq n$ | |
there are three possible cases, namely (1) $p_k=0$ for all $0\leq | |
k\leq n$ so that $z_m=z_n=w$, (2) $p_k=0$ for all $0\leq k\leq m$ but | |
there is $m+1\leq k\leq n$ such that $p_k=1$, so that $z_m=w$ and | |
$z_n=v_k$ for the smallest such $k$, and (3) there is $0\leq k\leq m$ | |
such that $p_k=1$, so that $z_m=z_n=v_k$ for the smallest such $k$. | |
Cases (1) and (3) are trivial (we have $z_m=z_n$), and in case (2) we | |
have $|z_m-z_n| \leq |w-v_k| \leq 2^{-k}$ but $m\leq k\leq n$, so | |
$|z_m-z_n| \leq 2^{-m}$ as desired. This concludes the claim. | |
We have just seen that $(z_n)$ is Cauchy with modulus $|z_m-z_n| \leq | |
2^{-\min(m,n)}$. Therefore it has a limit $z$. Note that $\chi(z)=0$ | |
or $\chi(z)=1$. | |
If there is $k$ such that $p_k=1$ then $z_n$ is equal to $v_k$ for all | |
$n\geq k$ where $k$ is the smallest such $k$, and in particular, $z = | |
v_k$, which gives $\chi(z) = \chi(v_k) = 1$; so contrapositively, | |
$\chi(z) = 0$ implies that $(p_n)$ is identically $0$. But | |
conversely, if $(p_n)$ is identically $0$ then $(z_n)$ is the constant | |
sequence with value $w$ so $z=w$, which gives $\chi(z) = \chi(w) = 0$. | |
This proves that $(p_n)$ is the zero sequence if and only if $\chi(z) | |
= 0$. Therefore, $(p_n)$ is the zero sequence or it is not. | |
This gives $\mathbf{WLPO}_{\mathrm{seq}}$, as claimed. | |
\end{proof} | |
\begin{lem}\label{dec-r-and-sequential-wlpo-imply-analytic-wlpo} | |
$\mathbf{DEC}(\mathbb{R})$ and $\mathbf{WLPO}_{\mathrm{seq}}$ together | |
imply $\mathbf{WLPO}_{\mathbb{R}}$ | |
\end{lem} | |
\begin{proof} | |
Assume $\chi\colon\mathbb{R}\to\{0,1\}$ satisfies $\chi(0)=0$ and | |
$\chi(1)=1$. Replacing $\chi$ by $t \mapsto \chi(\inf(1,|t|))$, we | |
can assume that in fact $\chi(t)=1$ whenever $|t|\geq 1$. | |
Now let $c$ be a real. We wish to show that $c$ is zero or it is not. | |
Define a binary sequence $(p_n)$ by $p_n := \chi(2^n c)$. | |
If $c\apart 0$, that is $|c|>0$, then there is $n$ such that | |
$2^n\,|c|\geq 1$, so $p_n = 1$; so contrapositively, if $(p_n)$ is the | |
zero sequence then $c=0$. But conversely, if $c=0$ then $p_n$ is | |
trivially constantly equal to $0$. This proves that $(p_n)$ is the | |
zero sequence if and only if $c = 0$. Now | |
$\mathbf{WLPO}_{\mathrm{seq}}$ tells us that $(p_n)$ is the zero | |
sequence or it is not. So we conclude that $z$ is zero or it is not. | |
This gives $\mathbf{WLPO}_{\mathbb{R}}$, as claimed. | |
\end{proof} | |
\thingy\label{sequential-wlpo-does-not-imply-analytic-wlpo} We have | |
remarked in ¶\ref{analytic-wlpo-implies-sequential-wlpo} that | |
$\mathbf{WLPO}_{\mathbb{R}} \; \Longrightarrow \; | |
\mathbf{WLPO}_{\mathrm{seq}}$. The converse generally does not hold. | |
To see this, (working in a classical universe) we consider the | |
topological model over $\mathbb{R}$, i.e., the topos of sheaves over | |
$\mathbb{R}$. | |
In this model, the statement $\mathbf{WLPO}_{\mathbb{R}}$ does not | |
hold, since it unwinds to saying that if $c$ is a continuous | |
real-valued function on an open subset $U$ of $\mathbb{R}$ then $U$ is | |
the union of the largest open set on which $c=0$ (i.e., the interior | |
of $\{x \in U : c(x) = 0\}$) and the largest open set on which $c\neq | |
0$ holds densely (i.e., the interior of the closure of $\{x \in U : | |
c(x) \neq 0\}$); but the function $\mathbb{R}\to\mathbb{R}$ given by | |
$x \mapsto \max(0,x)$ provides a counterexample to this. | |
However, $\mathbf{WLPO}_{\mathrm{seq}}$, and in fact even | |
$\mathbf{LPO}_{\mathrm{seq}}$, which asserts that every binary | |
sequence $(p_n)$ satisfies $\forall n.(p_n=0)$ or $\exists n.(p_n=1)$, | |
\emph{does} hold in the model. Indeed, $\mathbf{LPO}_{\mathrm{seq}}$ | |
unwinds to saying that if $U$ is an open subset of $\mathbb{R}$ and | |
$(p_n)$ a sequence of \emph{continuous} (= locally constant) functions | |
$U \to \{0,1\}$, then we can find open sets $V,W_n$ with $U = V\cup | |
\bigcup_n W_n$ such that all $p_n$ are identically zero on $V$ and | |
$p_n = 1$ on $W_n$. But now since $\mathbb{R}$ is locally connected, | |
so we can write $U$ as a union of connected open sets, and it is | |
enough to prove the statement for any one of these, so we can assume | |
$U$ connected. But when $U$ is connected, a locally constant function | |
$U \to \{0,1\}$ is constant, so each $p_n$ is just an element of | |
$\{0,1\}$ and the statement is trivial using the fact that the ambient | |
classical universe $\mathbf{LPO}_{\mathrm{seq}}$ holds trivially. | |
\thingy\label{sequential-wlpo-with-some-choice-implies-analytic-wlpo} | |
Despite ¶\ref{sequential-wlpo-does-not-imply-analytic-wlpo}, note that | |
a modicum of Choice suffices to give $\mathbf{WLPO}_{\mathbb{R}}$ from | |
$\mathbf{WLPO}_{\mathrm{seq}}$. Indeed, assume the latter, and assume | |
that Choice holds for sequences of inhabited subsets of $\{0,1\}$ | |
(that is: if $P_n \subseteq \{0,1\}$ and every $P_n$ is inhabited, | |
there exists $(p_n)$ such that $p_n \in P_n$ for all $n$). We wish to | |
show $\mathbf{WLPO}_{\mathbb{R}}$. | |
Let $c\in\mathbb{R}$. For every $n\in\mathbb{N}$, since $0 < 2^{-n}$, | |
we have either $|c| < 2^{-n}$ or $0 < |c|$. By the Choice principle | |
we assumed (precisely, letting $P_n := \{0 : |c| < 2^{-n}\} \cup \{1 : | |
0 < |c|\}$, which is inhabited), we can find a binary sequence $(p_n)$ | |
such that $|c| < 2^{-n}$ if $p_n=0$ and $0 < |c|$ if $p_n=1$. If | |
there is $n$ such that $p_n=1$ then $|c|>0$, that is $c\apart 0$; so | |
contrapositively, $c=0$ implies that $(p_n)$ is identically $0$. But | |
conversely, if $(p_n)$ is identically $0$ then $|c| < 2^{-n}$ for all | |
$n$ so that $c=0$. This shows that $c=0$ if and only if $(p_n)$ is | |
the zero sequence. Now $\mathbf{WLPO}_{\mathrm{seq}}$ tells us that | |
$(p_n)$ is the zero sequence or it is not. So we conclude that $z$ is | |
zero or it is not. This gives $\mathbf{WLPO}_{\mathbb{R}}$, as | |
claimed.\quad\qedsymbol | |
\hbox to\hsize{\hrulefill}\bigbreak | |
\thingy\label{wllpo-and-friends} Now let $\mathbb{R}\setminus\{0\} := | |
\{t \in \mathbb{R} : \neg(t=0)\}$, and consider the following | |
principles: | |
\begin{itemize} | |
\item “$\mathbf{DEC}(\mathbb{R}\setminus\{0\})$”: There exists subsets | |
$A,B \subseteq \mathbb{R}\setminus\{0\}$ such that $A$ and $B$ are | |
inhabited, $A \cap B = \varnothing$ and $A \cup B = | |
\mathbb{R}\setminus\{0\}$. | |
\item “$\mathbf{DEC'}(\mathbb{R}\setminus\{0\})$”: There exists | |
subsets $A,B \subseteq \mathbb{R}\setminus\{0\}$ such $-1\in A$ and | |
$1\in B$, and $A \cap B = \varnothing$ and $A \cup B = | |
\mathbb{R}\setminus\{0\}$. | |
\item “$\mathbf{WLLPO}_{\mathbb{R}}$” (also known as | |
$\mathbf{MP}^\vee_{\mathbb{R}}$): Every $c\in | |
\mathbb{R}\setminus\{0\}$ satisfies $c\leq 0$ or $c\geq 0$. | |
\item “$\mathbf{WLLPO}_{\mathrm{seq}}$” (also known as | |
$\mathbf{MP}^\vee_{\mathrm{seq}}$): If $(p_n)$ and $(q_n)$ are | |
binary sequences not both identically zero then one of them is not | |
identically zero (i.e., if $\neg(\forall n.(p_n=0) \land \forall | |
n.(q_n=0))$, then $\neg\forall n.(p_n=0)$ or $\neg\forall | |
n.(q_n=0)$). | |
\end{itemize} | |
\thingy Trivially, $\mathbf{DEC'}(\mathbb{R}\setminus\{0\}) \; | |
\Longrightarrow \; \mathbf{DEC}(\mathbb{R}\setminus\{0\})$. | |
As in ¶\ref{dec-vs-chi-function}, | |
$\mathbf{DEC}(\mathbb{R}\setminus\{0\})$ can be rephrased as the | |
existence of a function $\chi\colon\mathbb{R}\setminus\{0\}\to\{0,1\}$ | |
such that $\chi(a) = 0$ and $\chi(b) = 1$ for some $a$ and $b$ in | |
$\mathbb{R}\setminus\{0\}$. Unlike in ¶\ref{dec-vs-chi-function}, | |
however, there is no obvious way to assume, say, $a = -1$ and $b = 1$, | |
which is why we introduce the perhaps more natural | |
$\mathbf{DEC'}(\mathbb{R}\setminus\{0\})$, which can be rephrased as | |
the existence of a function | |
$\chi\colon\mathbb{R}\setminus\{0\}\to\{0,1\}$ such that $\chi(-1) = | |
0$ and $\chi(1) = 1$. We \emph{can}, however, do something with $b$: | |
\begin{lem}[Mohammad Tahmasbi]\label{dec-r0-reduction} | |
In the above formulation of $\mathbf{DEC}(\mathbb{R}\setminus\{0\})$, | |
we can additionally assume $a<b$ and $b=1$. In other words, | |
$\mathbf{DEC}(\mathbb{R}\setminus\{0\})$ is equivalent to the | |
statement that there exists a function | |
$\chi\colon\mathbb{R}\setminus\{0\}\to\{0,1\}$ such that $\chi(a) = 0$ | |
for some $a<1$ in $\mathbb{R}\setminus\{0\}$ and $\chi(1) = 1$. | |
\end{lem} | |
(To point out a tempting mistake which the author made at one point, | |
we cannot get $b=1$ straightforwardly by replacing $\chi$ by $t | |
\mapsto \chi(bt)$: this is indeed defined on | |
$\mathbb{R}\setminus\{0\}$ since $\neg(b=0)$ and $\neg(t=0)$ imply | |
$\neg(bt=0)$, but after this replacement is then no longer clear that | |
there exists $a'$ such that $\chi(a') = 0$, since we cannot | |
constructively assert that the equation $bt=a$ has a solution when | |
$\neg(b=0)$. We need to proceed with more care in the following | |
proof.) | |
\begin{proof}[Proof of lemma \ref{dec-r0-reduction}] | |
We assume the existence of a function | |
$\chi\colon\mathbb{R}\setminus\{0\}\to\{0,1\}$ such that $\chi(a) = 0$ | |
and $\chi(b) = 1$ for some $a$ and $b$ in $\mathbb{R}\setminus\{0\}$, | |
and our goal is to achieve the same conclusion with the additional | |
constraints $a<b$ and $b=1$. | |
Let us first achieve $a<b$ (still with $\chi(a)=0$ and $\chi(b)=1$). | |
To this effect, note that either $a<b$ or $b<a+1$. In the first case | |
we are done. In the latter case, note that $a+1<0$ or $0<a+2$. In | |
the subcase $b<a+1<0$ (note in particular $\neg(a+1=0)$), let us | |
consider the value $\chi(a+1)$, which is either $0$ or $1$. In the | |
sub-subcase $\chi(a+1)=0$, we redefine $a,b,\chi$ to be $b,a+1,1-\chi$ | |
so we have achieved the required constraint; in the sub-subcase | |
$\chi(a+1)=1$, we redefine $a,b$ to be $a,a+1$ so we have achieved the | |
required constraint. In the subcase $0<a+2$ (note in particular | |
$\neg(a+2=0)$), we similarly consider the value $\chi(a+2)$, which is | |
either $0$ or $1$. In the sub-subcase $\chi(a+2)=0$, we redefine | |
$a,b,\chi$ to be $b,a+2,1-\chi$ so we have achieved the required | |
constraint (since $b<a+1<a+2$); in the sub-subcase $\chi(a+2)=1$, we | |
redefine $a,b$ to be $a,a+2$ so we have achieved the required | |
constraint. Thus, we have proved in all cases the existence of a | |
function $\chi\colon\mathbb{R}\setminus\{0\}\to\{0,1\}$ such that | |
$\chi(a) = 0$ and $\chi(b) = 1$ for some $a<b$ in | |
$\mathbb{R}\setminus\{0\}$. | |
Now let us additionally achieve $b=1$ (still with $\chi(a)=0$ and | |
$\chi(b)=1$ and $a<b$). Since $a<b$ we have either $0<b$ or $a<0$. | |
In the first case we redefine $a,b,\chi$ to be $a/b$, $1$ and | |
$t\mapsto \chi(bt)$ (note that $b$ is invertible as $b>0$) to obtain | |
the additional constraint $b=1$. In the second case, we first | |
redefine $a,b,\chi$ to be $-b$, $-a$ and $t\mapsto 1-\chi(-t)$, which | |
reduces us to the first case. Thus, we have proved in all cases the | |
existence of a function $\chi\colon\mathbb{R}\setminus\{0\}\to\{0,1\}$ | |
such that $\chi(a) = 0$ and $\chi(1) = 1$ for some $a<1$ in | |
$\mathbb{R}\setminus\{0\}$. | |
\end{proof} | |
Maybe the following could come in useful: | |
\begin{lem}\label{dec-r0-disjunction} | |
The statement $\mathbf{DEC}(\mathbb{R}\setminus\{0\})$ is equivalent | |
to the statement that | |
\begin{itemize}\setlength\itemsep{0pt} | |
\item\emph{either} $\mathbf{DEC'}(\mathbb{R}\setminus\{0\})$ holds, | |
that is, there exists $\chi\colon\mathbb{R}\setminus\{0\}\to\{0,1\}$ | |
such that $\chi(-1) = 0$ and $\chi(1) = 1$, | |
\item\emph{or} there exists | |
$\chi\colon\mathbb{R}\setminus\{0\}\to\{0,1\}$ such that $\chi(1) = | |
\chi(-1) = 1$ and $\chi(a) = \chi(-a) = 0$ for some $a \in | |
\mathbb{R}\setminus\{0\}$ (we could call this | |
$\mathbf{DEC''}(\mathbb{R}\setminus\{0\})$). | |
\end{itemize} | |
\end{lem} | |
\begin{proof} | |
It is trivial that each of the disjuncts | |
$\mathbf{DEC'}(\mathbb{R}\setminus\{0\})$ and | |
$\mathbf{DEC''}(\mathbb{R}\setminus\{0\})$ implies | |
$\mathbf{DEC}(\mathbb{R}\setminus\{0\})$. So we are to prove the | |
converse. | |
We have just seen in the previous lemma that | |
$\mathbf{DEC}(\mathbb{R}\setminus\{0\})$ is equivalent to the | |
statement that there exists a function | |
$\chi\colon\mathbb{R}\setminus\{0\}\to\{0,1\}$ such that $\chi(a) = 0$ | |
for some $a<1$ in $\mathbb{R}\setminus\{0\}$ and $\chi(1) = 1$, so we | |
assume this. | |
Now consider $\chi(-1)$. It is either $0$ or $1$. If it is $0$ then | |
the first disjunct ($\mathbf{DEC'}(\mathbb{R}\setminus\{0\})$) holds | |
trivially, and we are done. If it is $1$, consider $\chi(-a)$ (noting | |
that $\neg(-a=0)$ since $\neg(a=0)$). It is either $0$ or $1$. If it | |
is $0$ then the second disjunct | |
($\mathbf{DEC''}(\mathbb{R}\setminus\{0\})$) holds trivially, and we | |
are done. Finally, if $\chi(-a)=1$, then replacing $\chi$ by $t | |
\mapsto \chi(-at)$ brings us again to the first disjunct | |
($\mathbf{DEC'}(\mathbb{R}\setminus\{0\})$). So in any case we have | |
$\mathbf{DEC'}(\mathbb{R}\setminus\{0\})$ or | |
$\mathbf{DEC''}(\mathbb{R}\setminus\{0\})$, and this concludes the | |
proof. | |
\end{proof} | |
(To point out another tempting mistake which the author made at one | |
point, in $\mathbf{DEC''}(\mathbb{R}\setminus\{0\})$, the fact that | |
$\chi(a) = \chi(-a) = 0$ need not entail $\chi(|a|) = 0$. If we | |
\emph{had} $\chi(|a|) = 0$ then we could redefine $\chi$ as $t \mapsto | |
\chi(\inf(1, |a| + |t|))$ to obtain $\mathbf{DEC}(\mathbb{R})$ | |
from ¶\ref{wlpo-and-friends}–\ref{dec-vs-chi-function}, from which we | |
already know from theorem \ref{dec-r-equivalent-to-analytic-wlpo} that | |
$\mathbf{WLPO}_{\mathbb{R}}$ and in particular | |
$\mathbf{WLLPO}_{\mathbb{R}}$ follows as explained | |
in ¶\ref{analytic-wlpo-implies-analytic-wllpo}. Alas!, one cannot | |
constructively conclude $\chi(|a|) = 0$ from $\chi(a) = \chi(-a) = 0$ | |
unless one somehow already knew $\mathbf{WLLPO}_{\mathbb{R}}$.) | |
\thingy\label{analytic-wlpo-implies-analytic-wllpo} We point out the | |
standard fact that $\mathbf{WLPO}_{\mathbb{R}} \; \Longrightarrow \; | |
\mathbf{WLLPO}_{\mathbb{R}}$. To see this, assume | |
$\mathbf{WLPO}_{\mathbb{R}}$ holds, and let $c \in | |
\mathbb{R}\setminus\{0\}$. By $\mathbf{WLPO}_{\mathbb{R}}$, we have | |
either $\sup(x,0) = 0$ or $\neg(\sup(x,0) = 0)$. Now $x\leq 0$ is | |
equivalent to $\sup(x,0) = 0$, so this says that $x\leq 0$ or | |
$\neg(x\leq 0)$. But the latter implies $x\geq 0$ (because $x<0$ | |
implies $x\leq 0$, so contrapositively, $\neg(x\leq 0)$ implies | |
$\neg(x<0)$ which is just $x\geq 0$). So we have proved $x\leq 0$ or | |
$x\geq 0$. This gives $\mathbf{WLLPO}_{\mathbb{R}}$.\quad\qedsymbol | |
\thingy\label{sequential-wlpo-implies-sequential-wllpo} Let us | |
similarly point out that $\mathbf{WLPO}_{\mathrm{seq}} \; | |
\Longrightarrow \; \mathbf{WLLPO}_{\mathrm{seq}}$. To see this, | |
assume $\mathbf{WLPO}_{\mathrm{seq}}$ holds, and let $(p_n)$ and | |
$(q_n)$ be binary sequences such that $\neg(\forall n.(p_n=0) \land | |
\forall n.(q_n=0))$. By $\mathbf{WLPO}_{\mathrm{seq}}$, we have | |
either $\forall n.(p_n=0)$ or $\neg\forall n.(p_n=0)$, and similarly | |
either $\forall n.(q_n=0)$ or $\neg\forall n.(q_n=0)$. This gives us | |
four cases: the case where $\forall n.(p_n=0)$ and $\forall n.(q_n=0)$ | |
is ruled out by our assumption, and in all three remaining cases, the | |
conclusion that $\neg\forall n.(p_n=0)$ or $\neg\forall n.(q_n=0)$ is | |
immediate. So we have proved | |
$\mathbf{WLLPO}_{\mathrm{seq}}$.\quad\qedsymbol | |
\thingy There are various standard equivalent formulations of | |
$\mathbf{WLLPO}_{\mathrm{seq}}$. Rather than using two binary | |
sequences $(p_n)$ and $(q_n)$, it is sometimes more convenient to | |
group them into a single one by $s_{2n} = p_n$ and $s_{2n+1} = q_n$; | |
the statement $\mathbf{WLLPO}_{\mathrm{seq}}$ is then seen to be | |
equivalent to the folllowing: “if a binary sequence is not identically | |
zero then either its even terms are not identically zero or its odd | |
terms are not identically zero”. | |
Furthermore, it is sometimes convenient to replace $(s_m)$ by the | |
sequence whose $m$-th term equals $s_m$ except if there exists $k<m$ | |
such that $s_k=1$ in which case it equals $0$ (so that $(s_m)$ has at | |
most one term equal to $1$). This shows that in | |
$\mathbf{WLLPO}_{\mathrm{seq}}$ we can assume that $(p_n)$ and $(q_n)$ | |
jointly\footnote{To avoid any possible doubt, the clause “the binary | |
sequences $(p_n)$ and $(q_n)$ jointly have at most one term equal | |
to $1$” means that if one term of one of the sequences is $1$ then | |
every other term of that sequence and every term of the other | |
sequence are $0$.} have at most one term equal to $1$ (as well as | |
not being both zero); furthermore, the conclusion can then be stated | |
as the fact that either $(p_n)$ is the zero sequence or $(q_n)$ is so. | |
In other words, $\mathbf{WLLPO}_{\mathrm{seq}}$ is equivalent to: “if | |
two binary sequences jointly have at most one term equal to $1$ and if | |
they are not both identically zero, then one of them is identically | |
zero”. | |
\thingy\label{analytic-wllpo-implies-sequential-wllpo} Similarly | |
to ¶\ref{analytic-wlpo-implies-sequential-wlpo}, let us prove the | |
standard fact that $\mathbf{WLLPO}_{\mathbb{R}}$ implies | |
$\mathbf{WLLPO}_{\mathrm{seq}}$. If $(p_n)$ and $(q_n)$ are binary | |
sequence which jointly have at most one term equal to $1$ and which | |
are not both identically zero, we define a real sequence $z_n := | |
\sum_{k=0}^n (p_k-q_k)\cdot 2^{-k}$. Clearly this sequence is Cauchy | |
with modulus $|z_m-z_n| \leq 2^{-\min(m,n)}$, so it has a limit $z$. | |
If there is $n$ such that $p_n = 1$, then $z = 2^{-n}$, so $z > 0$, | |
and if there is $n$ such that $q_n = 1$, then $z = -2^{-n}$, so $z < | |
0$; so contrapositively, $z\leq 0$ implies that $(p_n)$ is | |
identically $0$, and $z\geq 0$ implies that $(q_n)$ is | |
identically $0$. But conversely, if $(p_n)$ is identically $0$ then | |
trivially $z\leq 0$, and if $(q_n)$ is identically $0$ then trivially | |
$z\geq 0$. This shows that $(p_n)$ is the zero sequence if and only | |
if $z\leq 0$, and $(q_n)$ is the zero sequence if and only if $z\geq | |
0$. Now $\mathbf{WLLPO}_{\mathbb{R}}$ tells us that $\neg(z=0)$ | |
implies $z\leq 0$ or $z\geq 0$. So we conclude that $(p_n)$ is the | |
zero sequence or it is not. This gives | |
$\mathbf{WLLPO}_{\mathrm{seq}}$.\quad\qedsymbol | |
\thingy As an aside, to see that the converse of | |
$\mathbf{WLLPO}_{\mathbb{R}} \; \Longrightarrow \; | |
\mathbf{WLLPO}_{\mathrm{seq}}$ generally does not hold, we use the | |
same topological model as | |
in ¶\ref{sequential-wlpo-does-not-imply-analytic-wlpo}, i.e., the | |
topos of sheaves over $\mathbb{R}$. We have already pointed out there | |
that it satisfies $\mathbf{WLPO}_{\mathrm{seq}}$ (which is stronger | |
than $\mathbf{WLLPO}_{\mathrm{seq}}$ | |
per ¶\ref{sequential-wlpo-implies-sequential-wllpo}). | |
But even $\mathbf{WLLPO}_{\mathbb{R}}$ does not hold in the model, | |
since it unwinds to saying that if $c$ is a continuous real-valued | |
function on an open subset $U$ of $\mathbb{R}$ which vanishes on no | |
nonempty open subset of $U$ then $U$ is the union of an open set on | |
which $c\leq 0$ one on which $c\geq 0$; but the function | |
$\mathbb{R}\to\mathbb{R}$ given by $x \mapsto x$ provides a | |
counterexample to this. | |
\thingy The implication $\mathbf{WLLPO}_{\mathbb{R}} \; | |
\Longrightarrow \; \mathbf{DEC'}(\mathbb{R}\setminus\{0\})$ is | |
trivial. Indeed, $\mathbf{WLLPO}_{\mathbb{R}}$ asserts precisely that | |
$\mathbb{R}\setminus\{0\}$ is the disjoint union of $\{t : t\gtrdot | |
0\}$ and $\{t : t\lessdot 0\}$ (here we write $x\gtrdot y$ for $(x\geq | |
y) \land \neg (x=y)$, or equivalently, $\neg(x\leq y)$), with of | |
course $1\gtrdot 0$ and $-1\lessdot 0$. | |
\thingy To summarize what has been said, the following implications | |
hold: | |
\begin{center} | |
\begin{tikzpicture} | |
\matrix(diag)[matrix of math nodes,column sep=2em,row sep=2ex]{ | |
\mathbf{WLPO}_{\mathbb{R}} | |
&\mathbf{WLLPO}_{\mathbb{R}} | |
&\mathbf{DEC'}(\mathbb{R}\setminus\{0\}) | |
&\mathbf{DEC}(\mathbb{R}\setminus\{0\})\\ | |
\mathbf{WLPO}_{\mathrm{seq}}&\mathbf{WLLPO}_{\mathrm{seq}}&&\\}; | |
\draw[->,double equal sign distance, -{implies[scale=1.5]}] (diag-1-1) -- (diag-1-2); | |
\draw[->,double equal sign distance, -{implies[scale=1.5]}] (diag-1-2) -- (diag-1-3); | |
\draw[->,double equal sign distance, -{implies[scale=1.5]}] (diag-1-3) -- (diag-1-4); | |
\draw[->,double equal sign distance, -{implies[scale=1.5]}] (diag-2-1) -- (diag-2-2); | |
\draw[->,double equal sign distance, -{implies[scale=1.5]}] (diag-1-1) -- (diag-2-1); | |
\draw[->,double equal sign distance, -{implies[scale=1.5]}] (diag-1-2) -- (diag-2-2); | |
\end{tikzpicture} | |
\end{center} | |
Neither of the vertical arrows is reversible in general. | |
Countermodels showing that the converse of both $\mathbf{WLPO} \; | |
\Longrightarrow \; \mathbf{WLLPO}$ implications does not hold in | |
general are also well known: Hyland's effective topos, for example, | |
satisfies all forms of $\mathbf{WLLPO}$ (as they follow from Markov's | |
principle) but no form of $\mathbf{WLPO}$: we do not elaborate here as | |
this is not our focus. | |
\begin{ques}\label{dec-r0-versus-wllpo-question} | |
Is $\mathbf{DEC}(\mathbb{R}\setminus\{0\})$ equivalent to | |
$\mathbf{WLLPO}_{\mathbb{R}}$? Is | |
$\mathbf{DEC'}(\mathbb{R}\setminus\{0\})$ equivalent to | |
$\mathbf{WLLPO}_{\mathbb{R}}$? | |
\end{ques} | |
We at least have the following partial analogue of | |
lemma \ref{dec-r-implies-sequential-wlpo}: | |
\begin{lem}\label{decprime-r0-implies-sequential-wllpo} | |
$\mathbf{DEC'}(\mathbb{R}\setminus\{0\})$ implies $\mathbf{WLLPO}_{\mathrm{seq}}$ | |
\end{lem} | |
\begin{proof} | |
Assume $\chi\colon\mathbb{R}\setminus\{0\}\to\{0,1\}$ satisfies | |
$\chi(-1)=0$ and $\chi(1)=1$. | |
Define real sequences $(u_n)$ and $(v_n)$ by induction as follows: put | |
$u_0=-1$ and $v_0=1$, and then : | |
\begin{itemize}\setlength\itemsep{0pt} | |
\item when $\chi(\frac{u_n+2v_n}{3})=0$ let $u_{n+1} = | |
\frac{u_n+2v_n}{3}$ and $v_{n+1} = v_n$, | |
\item when $\chi(\frac{u_n+2v_n}{3})=1$ and | |
$\chi(\frac{2u_n+v_n}{3})=1$ let $u_{n+1} = u_n$ and | |
$v_{n+1} = \frac{2u_n+v_n}{3}$, | |
\item when $\chi(\frac{u_n+2v_n}{3})=1$ and | |
$\chi(\frac{2u_n+v_n}{3})=0$ let $u_{n+1} = \frac{2u_n+v_n}{3}$ and | |
$v_{n+1} = \frac{u_n+2v_n}{2}$. | |
\end{itemize} | |
It is easy to see inductively that $(u_n)$ and $(v_n)$ are nonzero | |
rationals (with denominator dividing $3^n$): indeed, considering the | |
construction up to $n$, so long as the third case happens we have $u_k | |
= -3^{-k}$ and $v_k = 3^{-k}$, and as soon as the first (resp. second) | |
case happens we have $u_n,v_n>0$ (resp. $u_n,v_n<0$) from then on. | |
This justifies that the construction can be carried out (the function | |
$\chi$ is evaluated on nonzero rationals). | |
Obviously we have $\chi(u_n)=0$ and $\chi(v_n)=1$ for all $n$ by | |
induction. Furthermore, $(u_n)$ and $(v_n)$ satisfy $-1\leq u_m\leq | |
u_n\leq v_n\leq v_m\leq 1$ when $m\leq n$, and $|u_n-v_n| \leq | |
3^{-n}$. In particular, they are Cauchy with modulus $|u_m-u_n| \leq | |
3^{-\min(m,n)}$ and $|v_m-v_n| \leq 3^{-\min(m,n)}$, and they have the | |
same limit. Call $w$ this common limit: we have $|w-u_n| \leq 3^{-n}$ | |
and $|w-v_n| \leq 3^{-n}$. | |
Now let $(p_n)$ and $(q_n)$ be binary sequences which jointly have at | |
most one term equal to $1$ and which are not both identically zero. | |
We wish to show that $(p_n)$ is the zero sequence or that $(q_n)$ is | |
the zero sequence. | |
Define a real sequence $(z_n)$ as follows: | |
\begin{itemize}\setlength\itemsep{0pt} | |
\item when $p_k=0$ and $q_k=0$ for all $0\leq k\leq n$, let $z_n = w$, | |
\item when there exists an $0\leq k\leq n$ (necessarily unique) such | |
that $p_k=1$, let $z_n = v_k$, | |
\item when there exists an $0\leq k\leq n$ (necessarily unique) such | |
that $q_k=1$, let $z_n = u_k$. | |
\end{itemize} | |
(In other words, $z_n$ is constantly $w$ until such a value as $p_k=1$ | |
or $q_k=1$ if it is found, and is then equal to the corresponding | |
value of $v_k$ or $u_k$. This makes sense as we have assumed that | |
$(p_n)$ and $(q_n)$ jointly have at most one term equal to $1$.) This | |
sequence is well-defined. | |
By an easy case distinction as in the proof of | |
lemma \ref{dec-r-implies-sequential-wlpo}, we have that $(z_n)$ is | |
Cauchy with modulus $|z_m-z_n| \leq 3^{-\min(m,n)}$. Therefore it has | |
a limit $z$. | |
If there is $k$ such that $p_k=1$ then $z_n$ is equal to $v_k$ for all | |
$n\geq k$, and in particular, $z = v_k \apart 0$. Similarly, if there | |
is $k$ such that $q_k=1$ then $z_n$ is equal to $u_k$ for all $n\geq | |
k$, and in particular, $z = u_k \apart 0$. Contrapositively, if $z = | |
0$ then $(p_n)$ and $(q_n)$ are both identically zero; but we have | |
assumed that this be not the case: this shows $\neg(z=0)$. We are now | |
allowed to speak of $\chi(z)$. Note that then $\chi(z)=0$ or | |
$\chi(z)=1$. | |
Now if there is $k$ such that $p_k=1$ then $z = v_k$ so $\chi(z) = | |
\chi(v_k) = 1$; and if there is $k$ such that $q_k=1$ then $z = u_k$ | |
so $\chi(z) = \chi(u_k) = 0$. So contrapositively, $\chi(z) = 0$ | |
implies that $(p_n)$ is identically zero, and $\chi(z) = 1$ implies | |
that $(q_n)$ is identically zero. It follows that $(p_n)$ is the zero | |
sequence or that $(q_n)$ is the zero sequence. | |
This gives $\mathbf{WLLPO}_{\mathrm{seq}}$, as claimed. | |
\end{proof} | |
\thingy\label{sequential-wllpo-with-some-choice-implies-analytic-wllpo} | |
Like in ¶\ref{sequential-wlpo-with-some-choice-implies-analytic-wlpo}, | |
a modicum of Choice suffices to give $\mathbf{WLLPO}_{\mathbb{R}}$ | |
from $\mathbf{WLLPO}_{\mathrm{seq}}$. Indeed, assume again like there | |
that Choice holds for sequences of inhabited subsets of $\{0,1\}$, and | |
let us show that $\mathbf{WLLPO}_{\mathrm{seq}}$ implies | |
$\mathbf{WLLPO}_{\mathbb{R}}$ under this assumption. Let $c \in | |
\mathbb{R}\setminus\{0\}$. For any $n\in\mathbb{N}$ we have either | |
$c<2^{-n}$ or $c>0$. By the Choice principle we assumed (just like | |
in ¶\ref{sequential-wlpo-with-some-choice-implies-analytic-wlpo}), we | |
can find binary sequence $(p_n)$ such that $c<2^{-n}$ if $p_n=0$ and | |
$c>0$ if $p_n=1$; similarly, we can find $(q_n)$ such that $c>-2^{-n}$ | |
if $q_n=0$ and $c<0$ if $q_n=1$. Now if there is $n$ such that | |
$p_n=1$ then $c>0$, so contrapositively, $c\leq 0$ implies that | |
$(p_n)$ is identically $0$. But conversely, if $(p_n)$ is | |
identically $0$ then $c\leq 2^{-n}$ for all $n$ so that $c\leq 0$. | |
This shows that $c\leq 0$ if and only if $(p_n)$ is the zero sequence. | |
Analogously, $c\geq 0$ if and only if $(q_n)$ is the zero sequence. | |
Our assumption that $\neg(c=0)$ implies that $(p_n)$ and $(q_n)$ are | |
not both identically zero, so $\mathbf{WLLPO}_{\mathrm{seq}}$ tells us | |
that one of them is not identically zero, which implies that one of | |
$c\leq 0$ or $c\geq 0$ must be false, giving $c\gtrdot 0$ or | |
$c\lessdot 0$. This gives $\mathbf{WLLPO}_{\mathbb{R}}$, as claimed. | |
\thingy\emph{Comments and thoughts:} | |
The main issue in the question of whether | |
$\mathbf{DEC'}(\mathbb{R}\setminus\{0\})$ implies | |
$\mathbf{WLLPO}_{\mathbb{R}}$ is the absence of Choice. As explained | |
in ¶\ref{sequential-wllpo-with-some-choice-implies-analytic-wllpo}, if | |
we have a modicum of Choice, then $\mathbf{WLLPO}_{\mathrm{seq}}$ | |
implies $\mathbf{WLLPO}_{\mathbb{R}}$, and therefore | |
lemma \ref{decprime-r0-implies-sequential-wllpo} shows that | |
$\mathbf{DEC'}(\mathbb{R}\setminus\{0\})$ implies | |
$\mathbf{WLLPO}_{\mathbb{R}}$ under this assumption. | |
The idea to show that $\mathbf{DEC'}(\mathbb{R}\setminus\{0\})$ | |
implies $\mathbf{WLLPO}_{\mathbb{R}}$ without using Choice should be | |
to replace Choice by a judicious use of the $\chi$ function. Of | |
course we can replace $\chi$ by $t \mapsto \chi(\sup(-1,\inf(t,1)))$ | |
so as to get $\chi(t)=0$ if $t\leq -1$ and $\chi(t)=1$ if $t\geq 1$. | |
We would \emph{like} to do something like this: consider $\chi(2^{n+1} | |
c - 1)$: if it is $0$ then we know $c \leq 2^{-n}$ and we put $p_n = | |
0$ while if it is $1$ then we know $c \geq 0$ and we put $p_n = 1$; | |
and analogously, consider $\chi(2^{n+1} c + 1)$: if it is $1$ then we | |
know $c \geq -2^{-n}$ and we put $q_n = 0$ while if it is $0$ then we | |
know $c \leq 0$ and we put $q_n = 1$; if this could do this, the rest | |
of the proof of | |
¶\ref{sequential-wllpo-with-some-choice-implies-analytic-wllpo} would | |
work pretty much the same and we would get $c\leq 0$ or $c\geq 0$. | |
The problem with this “proof” is that we cannot consider $\chi(2^n c - | |
1)$ without first knowing that $\neg(c=2^{-n})$; and there does not | |
seem to be any way to attain such a non-equality without first being | |
able to separate $c$ from $0$ in some way. (Of course, one way to get | |
$\neg(c=2^{-n})$ is to have $\chi(c) \neq \chi(2^{-n})$ but there is | |
no reason to think the function $\chi$ would oblige us this way.) | |
Considering just $\chi(2^n c)$ is legitimate, and this gives us a | |
binary sequence $(r_n)$ which “eventually knows” whether $c\geq 0$ or | |
$c\leq 0$ in the sense that $r_n$ is eventually $0$ (in the sense: | |
$\exists n_0.\forall n\geq n_0.(r_n=0)$) if $c<0$, and eventually $1$ | |
if $c>0$. So we can deduce $\mathbf{WLLPO}_{\mathbb{R}}$ from | |
$\mathbf{DEC'}(\mathbb{R}\setminus\{0\})$ combined with the following | |
principle: “If $(r_n)$ is a binary sequence then either $(r_n)$ is not | |
eventually $0$ or $(r_n)$ is not eventually $1$.” However, it is not | |
hard to see that the quoted principle is, in fact, equivalent to | |
$\mathbf{WLPO}_{\mathrm{seq}}$. (Sketch of proof. If it holds, then | |
we get $\mathbf{WLPO}_{\mathrm{seq}}$ for a binary sequence $(p_n)$ by | |
defining $r_n$ to be $1$ if there is $0\leq k\leq n$ such that $p_n=1$ | |
and $0$ otherwise. Conversely, if $\mathbf{WLPO}_{\mathrm{seq}}$ | |
holds and if $(r_n)$ is a binary sequence, we first define $r'_n \in | |
\{0,1,{?}\}$ to be $0$ if $\forall m\geq n.(r_m=0)$, $1$ if $\forall | |
m\geq n.(r_m=1)$ and “${?}$” if neither holds: | |
$\mathbf{WLPO}_{\mathrm{seq}}$ ensures that this disjunction is | |
exhaustive, so that $r'_n$ makes sense; but further application of | |
$\mathbf{WLPO}_{\mathrm{seq}}$ allows us to conclude that either | |
$r'_n$ consists exclusively of ${?}$, or it is false that it does not | |
take the value $0$, or it is false that it does not take the value | |
$1$. In the first two cases, $(r_n)$ is not eventually $1$, and in | |
the third case, $(r_n)$ is not eventually $0$.~\qedsymbol) So this | |
shows that $\mathbf{DEC'}(\mathbb{R}\setminus\{0\})$ and | |
$\mathbf{WLPO}_{\mathrm{seq}}$ together imply | |
$\mathbf{WLLPO}_{\mathbb{R}}$, which is a far cry from answering even | |
the second part of question \ref{dec-r0-versus-wllpo-question}. | |
\thingy\emph{More comments and thoughts:} | |
Related to the question of whether | |
$\mathbf{DEC'}(\mathbb{R}\setminus\{0\})$ implies | |
$\mathbf{WLLPO}_{\mathbb{R}}$, we might consider the following | |
(somewhat loosely defined) \emph{game}, in the context of classical | |
mathematics: | |
Alice has hidden a nonzero real number $c$ in a black box, and Bob's | |
goal is to decide whether $c\leq 0$ or whether $c\geq 0$ (of course, | |
since we are talking classically here, exactly one of the two holds, | |
and we can write them as $c<0$ resp. $c>0$). Alice has also chosen a | |
function $\chi\colon\mathbb{R}\setminus\{0\}\to\{0,1\}$ such that | |
$\chi(t)=0$ if $t\leq-1$ and $\chi(t)=1$ if $t\geq 1$ (or maybe she | |
doesn't need to choose it aforehand, but every time it is evaluated | |
she must commit to a value and cannot change it afterwards). Bob can | |
ask Alice to perform certain computations on numbers hidden in boxes, | |
such as sums, differences, products, binary $\max$ and $\min$ (or even | |
computing the inverse of $x$ in a box if Bob can also produce a | |
positive rational lower bound on $|x|$): Alice will do these | |
computations for him, and return the result still in a box (and of | |
course she also will willingly put an explicitly given costant in a | |
box). Furthermore, Bob can ask Alice to evaluate $\chi$ on the | |
content of a box (or on an explicitly given number): in this case, Bob | |
gets to see the actual result, $0$ or $1$. But Bob can only ask for | |
the evaluation of $\chi$ provided he is certain (i.e., can prove) that | |
the input is $\neq 0$. Does Bob have a deterministic strategy that | |
guarantees success in finite time? | |
(As explained in the previous paragraphs, if we remove the restriction | |
that $\chi$ can only be evaluated on inputs $\neq 0$, then Bob has a | |
strategy that consists of computing $\chi(2^{n+1} c - 1)$ and | |
$\chi(2^{n+1} c + 1)$ until one of the former returns $1$ or one of | |
the latter returns $0$, which is bound to happen in finite time. And | |
with the restriction on $\chi$ but granting Bob a form of omniscience, | |
he can also do it by evaluating $\chi(2^n c)$ and divining whether it | |
is eventually $0$ or eventually $1$.) | |
I \emph{suspect} that Bob does not have a winning strategy for this | |
game (intuitively, because we can only get the required information | |
that the input of $\chi$ is nonzero by knowing something which makes | |
the output of $\chi$ useless; also, because any deterministic strategy | |
of Bob must apply to the particular case where $\chi$ is $1$ on the | |
positives and $0$ on the negatives, and then everything is continuous | |
in $c$), but I have been unable to actually prove it. | |
In any case, the exact relation between the (classical mathematics) | |
game I just defined and the question of whether | |
$\mathbf{DEC'}(\mathbb{R}\setminus\{0\})$ implies | |
$\mathbf{WLLPO}_{\mathbb{R}}$ constructively is also unclear. A | |
winning strategy for the game should probably give a proof of | |
$\mathbf{WLLPO}_{\mathbb{R}}$ from | |
$\mathbf{DEC'}(\mathbb{R}\setminus\{0\})$ together with perhaps | |
Markov's principle (which is stronger than | |
$\mathbf{WLLPO}_{\mathrm{seq}}$ but hopefully not too distant). But | |
conversely, whether a constructive implication from | |
$\mathbf{DEC'}(\mathbb{R}\setminus\{0\})$ to | |
$\mathbf{WLLPO}_{\mathbb{R}}$ would necessarily come in the form of a | |
winning strategy to this game is highly unclear to me: there are, | |
after all, other ways to define a real number constructively, such as | |
by the limit of a sequence or by a Dedekind cut (and it could be | |
important that we are working with Dedekind reals so maybe we | |
\emph{cannot} do things with sequences, let alone Cauchy sequences | |
having a modulus of convergence; but how does one usefully produce | |
Dedekind cuts from the function $\chi$?). | |
Maybe one should start by combining the proofs of lemmate | |
\ref{dec-r-implies-sequential-wlpo} and \ref{dec-r-and-sequential-wlpo-imply-analytic-wlpo} | |
into a single one as a source of inspiration on how to deal with the | |
analogous problem for $\mathbf{WLLPO}_{\mathbb{R}}$. | |
Another possible idea might be to try deducing | |
$\mathbf{WLLPO}_{\mathbb{R}}$ from | |
$\mathbf{DEC'}(\mathbb{R}\setminus\{0\})$ in a manner similar to | |
lemma \ref{decprime-r0-implies-sequential-wllpo} but working directly | |
from signed-bit representations as defined in Lubarsky \& Richman, | |
“Signed-Bit Representations of Real Numbers”, \textit{J. Log. Anal.} | |
\textbf{1} (2009), paper 10 (esp. theorems 2.3 \& 2.4), or | |
alternatively some kind of multivalued\footnote{It seems to be part of | |
the folklore that some kind of multivalued Cauchy reals are equivalent | |
to Dedekind reals, but references for this are sketchy at best.} | |
Cauchy sequence: instead of combining \emph{sequences} $(p_n)$ and | |
$(q_n)$ with the \emph{sequences} $(u_n)$ and $(v_n)$ (and $w$) to | |
produce a \emph{sequence} $(z_n)$ which converges to a limit we can | |
disjunct on, we would rather use some kind of multivalued sequence in | |
several of these places. But I have been unable to do this. | |
\end{document} |
Sign up for free
to join this conversation on GitHub.
Already have an account?
Sign in to comment
See https://mathoverflow.net/questions/479398/does-the-decomposability-of-mathbbr-imply-analytic-llpo/479501#479501 for context.